Stay ahead of learning milestones! Enroll in a class over the summer!

G
Topic
First Poster
Last Poster
Preparing for Putnam level entrance examinations
Cats_on_a_computer   4
N 2 hours ago by Cats_on_a_computer
Non American high schooler in the equivalent of grade 12 here. Where I live, two the best undergraduates program in the country accepts students based on a common entrance exam. The first half of the exam is “screening”, with 4 options being presented per question, each of which one has to assign a True or False. This first half is about the difficulty of an average AIME, or JEE Adv paper, and it is a requirement for any candidate to achieve at least 24/40 on this half for the examiners to even consider grading the second part. The second part consists of long form questions, and I have, no joke, seen them literally rip off, verbatim, Putnam A6s. Some of the problems are generally standard textbook problems in certain undergrad courses but obviously that doesn’t translate it to being doable for high school students. I’ve effectively got to prepare for a slightly nerfed Putnam, if you will, and so I’ve been looking for resources (not just problems) for Putnam level questions. Does anyone have any suggestions?
4 replies
Cats_on_a_computer
Yesterday at 8:32 AM
Cats_on_a_computer
2 hours ago
Marginal Profit
NC4723   1
N 5 hours ago by Juno_34
Please help me solve this
1 reply
NC4723
Dec 11, 2015
Juno_34
5 hours ago
Romania NMO 2023 Grade 11 P1
DanDumitrescu   15
N Today at 5:46 AM by anudeep
Source: Romania National Olympiad 2023
Determine twice differentiable functions $f: \mathbb{R} \rightarrow \mathbb{R}$ which verify relation

\[
    \left( f'(x) \right)^2 + f''(x) \leq 0, \forall x \in \mathbb{R}.
    \]
15 replies
DanDumitrescu
Apr 14, 2023
anudeep
Today at 5:46 AM
Subset Ordered Pairs of {1, 2, ..., 10}
ahaanomegas   11
N Today at 5:27 AM by cappucher
Source: Putnam 1990 A6
If $X$ is a finite set, let $X$ denote the number of elements in $X$. Call an ordered pair $(S,T)$ of subsets of $ \{ 1, 2, \cdots, n \} $ $ \emph {admissible} $ if $ s > |T| $ for each $ s \in S $, and $ t > |S| $ for each $ t \in T $. How many admissible ordered pairs of subsets $ \{ 1, 2, \cdots, 10 \} $ are there? Prove your answer.
11 replies
ahaanomegas
Jul 12, 2013
cappucher
Today at 5:27 AM
Putnam 2000 B4
ahaanomegas   6
N Today at 1:53 AM by mqoi_KOLA
Let $f(x)$ be a continuous function such that $f(2x^2-1)=2xf(x)$ for all $x$. Show that $f(x)=0$ for $-1\le x \le 1$.
6 replies
ahaanomegas
Sep 6, 2011
mqoi_KOLA
Today at 1:53 AM
Another integral limit
RobertRogo   1
N Yesterday at 8:37 PM by alexheinis
Source: "Traian Lalescu" student contest 2025, Section A, Problem 3
Let $f \colon [0, \infty) \to \mathbb{R}$ be a function differentiable at 0 with $f(0) = 0$. Find
$$\lim_{n \to \infty} \frac{1}{n} \int_{2^n}^{2^{n+1}} f\left(\frac{\ln x}{x}\right) dx$$
1 reply
RobertRogo
Yesterday at 2:28 PM
alexheinis
Yesterday at 8:37 PM
AB=BA if A-nilpotent
KevinDB17   3
N Yesterday at 7:51 PM by loup blanc
Let A,B 2 complex n*n matrices such that AB+I=A+B+BA
If A is nilpotent prove that AB=BA
3 replies
KevinDB17
Mar 30, 2025
loup blanc
Yesterday at 7:51 PM
Very nice equivalence in matrix equations
RobertRogo   3
N Yesterday at 5:45 PM by Etkan
Source: "Traian Lalescu" student contest 2025, Section A, Problem 4
Let $A, B \in \mathcal{M}_n(\mathbb{C})$ Show that the following statements are equivalent:

i) For every $C \in \mathcal{M}_n(\mathbb{C})$ there exist $X, Y \in \mathcal{M}_n(\mathbb{C})$ such that $AX + YB = C$
ii) For every $C \in \mathcal{M}_n(\mathbb{C})$ there exist $U, V \in \mathcal{M}_n(\mathbb{C})$ such that $A^2 U + V B^2 = C$

3 replies
RobertRogo
Yesterday at 2:34 PM
Etkan
Yesterday at 5:45 PM
Miklos Schweitzer 1971_5
ehsan2004   2
N Yesterday at 5:25 PM by pi_quadrat_sechstel
Let $ \lambda_1 \leq \lambda_2 \leq...$ be a positive sequence and let $ K$ be a constant such that \[  \sum_{k=1}^{n-1} \lambda^2_k < K \lambda^2_n \;(n=1,2,...).\] Prove that there exists a constant $ K'$ such that \[  \sum_{k=1}^{n-1} \lambda_k < K' \lambda_n \;(n=1,2,...).\]

L. Leindler
2 replies
ehsan2004
Oct 29, 2008
pi_quadrat_sechstel
Yesterday at 5:25 PM
Cute matrix equation
RobertRogo   1
N Yesterday at 4:46 PM by loup blanc
Source: "Traian Lalescu" student contest 2025, Section A, Problem 2
Find all matrices $A \in \mathcal{M}_n(\mathbb{Z})$ such that $$2025A^{2025}=A^{2024}+A^{2023}+\ldots+A$$
1 reply
RobertRogo
Yesterday at 2:23 PM
loup blanc
Yesterday at 4:46 PM
a